LSAT and Law School Admissions Forum

Get expert LSAT preparation and law school admissions advice from PowerScore Test Preparation.

User avatar
 Dave Killoran
PowerScore Staff
  • PowerScore Staff
  • Posts: 5852
  • Joined: Mar 25, 2011
|
#88555
Complete Question Explanation
(The complete setup for this game can be found here: lsat/viewtopic.php?f=182&p=88548#p88548)

The correct answer choice is (C).

Answer choice (A): This answer choice does not have to occur when Templates #1 and #2 are in effect.

Answer choice (B): This answer choice does not have to occur when Template #1 is in effect.

Answer choice (C): This is the correct answer choice. In every solution, H’s delivery is earlier than K’s delivery.

Answer choice (D): This answer choice does not have to occur when Template #1 is in effect.

Answer choice (E): This answer choice does not occur when Template #2 is in effect.
 kekethegreat
  • Posts: 5
  • Joined: Apr 14, 2021
|
#88476
How are we able to tell that H is before K? What do I need to know that tells me that it must be true? Thanks for the help in advance!
 Robert Carroll
PowerScore Staff
  • PowerScore Staff
  • Posts: 1787
  • Joined: Dec 06, 2013
|
#88484
keke,

The second rule requires H to be earlier than G. The first rule requires F to be earlier to G and G to be earlier than K. Isolate the parts of that sequence relevant to answer choice (C) for this question: H is earlier than G, and G is earlier than K. That's all we need to prove answer choice (C) here.

Robert Carroll

Get the most out of your LSAT Prep Plus subscription.

Analyze and track your performance with our Testing and Analytics Package.